The m∠ABC = (10x-5)°and m∠CBD=35°. If the angles are supplementary, find the value of x.

Answers

Answer 1

Answer:

15

Step-by-step explanation:

Supplementary angles completes each other to 180°

if the teo angles are supplementary here the the sum of 10x - 5 + 35 should be 180°

10x - 5 + 35 = 180

10x + 30 = 180

10x = 150

x = 15


Related Questions

What is the value of X ?

Answers

Answer:

D

Step-by-step explanation:

2² + 6² = x²

4 + 36 = x²

40 = x²

x = 2√10

Mr.Rice students ran a 40 yard dash in the following times 6.8,7.3,7.1 ,7.0,7.2,7.3,7.0 how many race times are recorded

Answers

the race times are recorded 7 times

The number of race times recorded as portrayed by the number of data points is seven(7).

What is the number of race times recorded for the dash?

From the task content;

It follows that the distance ran be Mr. Rice students was 40 yards.

Additionally, it follows from the task content that the times recorded were; 6.8,7.3,7.1 ,7.0,7.2,7.3 and 7.0.

On this note, the number of race times recorded as portrayed by the number of data points is seven(7).

Read more on data points;

https://brainly.com/question/3514929

a condition for two vectors to be equal is that?​

Answers

Answer:

Vector is equal to vector b. For two vectors to be equal, they must have both the magnitude and the directions equal.

Step-by-step explanation:

riley has a farm on a rectangular piece of land that is 200 meters wide

Answers

200 meters wide is the answer

Answer:

Do you mean "Riley has a farm on a rectangular piece of land that is  200   meters wide. This area is divided into two parts: A square area where she grows avocados (whose side is the same as the length of the farm), and the remaining area where she lives.  

Every week, Riley spends  $3  per square meter on the area where she lives, and earns  $7  per square meter from the area where she grows avocados. That way, she manages to save some money every week." ?

The answer is 7L^2>3l(200-l)

A sample of 899 Americans provides enough evidence to conclude that marketing campaign was effective. Provide a statement that should be put out by the marketing department. A. There is not sufficient evidence to conclude that the mean consumption of popcorn has risen. B. There is sufficient evidence to conclude that the mean consumption of popcorn has risen. C. There is sufficient evidence to conclude that the mean consumption of popcorn has stayed the same. D. There is not sufficient evidence to conclude that the mean consumption of popcorn has stayed the same.

Answers

Answer:

The correct answer to the following question will be Option A.

Step-by-step explanation:

Marketing Analyst seems to be responsible for information and evaluation that directs its marketing team and directs its marketing approach by defining the target clients as well as the competitiveness of the product.A survey of 899 American citizens requires appropriate evidence to demonstrate that perhaps the marketing strategy is working even though there was not considerable evidence to suggest that even the total demand for popcorn had increased.

Other given choices are not related to the given circumstances. So that option A seems to be the appropriate choice.

A sofa regularly sells for $450. The sale price is$337.50. Find the percent decrease of the sale price from the regular price

Answers

Answer:

25% decrease

Step-by-step explanation:

Take the original price and subtract the new price

450-337.50 =112.50

Divide by the original price

112.50/450=.25

Multiply by 100% to change to percent form

25%

A local country officials need to calculate the capacity of a large hole for the garbage refuse dump. The dump hole is 250 feet long,120 feet wide and 30 feet deep. What is the capacity of the dump hole in cubic feet.

Answers

Answer:

900000cubic feet

Step-by-step explanation:

capacity of dump hole= 250*120*30

= 900000cubic feet

What is the square root of 100?

Answers

Answer:

10

Step-by-step explanation:

Answer:

10

Step-by-step explanation:

Square root is finding what number times what gets your goal.

10 x 10 = 100 so 100 squared is 10.

5 x 5 = 25 so 25 squared is 5.

4 x 4 = 16 so 15 squared is 4.

You get it? :)

Have a nice day!

Please answer this correctly

Answers

Answer:

0-19: Make it 4 units tall

20-39: Make it 2 units tall

40-59: Make it 5 units tall

60-79: Make it 3 units tall

80-99: Make it 1 unit tall

Step-by-step explanation:

0-19: 4, 6, 19, 19 (4 numbers)

20-39: 29, 38 (2 numbers)

40-59: 40, 41, 41, 57, 58 (5 numbers)

60-79: 62, 66, 73 (3 numbers)

80-99: 87 (1 number)

Given the following information, find the probability that a randomly selected dog will be a golden retriever or a poodle. Number of dogs who are poodles: 31, golden retrievers: 58, beagles: 20, pugs: 38a) 39.5%b) 60.5%c) 58.0%d) 46.9%

Answers

Answer: a) 39.5%

Step-by-step explanation:

For random selections, we assume that all the dogs have the same probability of being selected.

In this case, the probability will be equal to the number of golden retrievers divided the total number of dogs.

We have 58 golden retrievers, and the total number of dogs is:

31 + 58 +20 + 38 = 147

Then the probability is:

P = 58/147 = 0.395

If we multiply it by 100%, we obtain the percentage form:

0.395*100% = 39.5%

So the correct option is a.

Use z scores to compare the given values. Based on sample​ data, newborn males have weights with a mean of 3259.6 g and a standard deviation of 722.4 g. Newborn females have weights with a mean of 3031.2 g and a standard deviation of 495.9 g. Who has the weight that is more extreme relative to the group from which they​ came: a male who weighs 1700 g or a female who weighs 1700 ​g? Since the z score for the male is zequals nothing and the z score for the female is zequals nothing​, the female female male has the weight that is more extreme.

Answers

Answer:

Since the z score for the male is z=-2.1589 and the z score for the female is z=-2.6844​, the female has the weight that is more extreme.

Step-by-step explanation:

To find the z score, we use the following equation:

[tex]z=\frac{x-m}{s}[/tex]

Where m is the mean and s is the standard deviation.

So, the z score for a male who weighs 1700 g is:

[tex]z=\frac{1700-3259.6}{722.4}=-2.1589[/tex]

At the same way, the z score for a female who weighs 1700 g is:

[tex]z=\frac{1700-3031.2}{495.9}=-2.6844[/tex]

Finally, -2.6844 is farther from zero than -2.1589, so the female has the weight that is more extreme.

Find the 1000th term for the sequence

Answers

Answer:

D. 7017

Step-by-step explanation:

if 24 is the first term, find 7x999, or 7x1000-7 and add 24

however a better way would be to use the formula

value=a+(n-1)d

a = the first term in the sequence (24)

n     =     the amount of terms you need (1000)

d = the common difference between terms (7)

I promise brainliest and a exter 25 poinst to the first to answer What is the solution to the inequality 2x ≥ -4? Click the number line until the correct answer is shown...

Answers

:

Answer:

the answer is the arrow going to the right because its not a negative number and a closed circle

Step-by-step explanation:

so that means that 2x is at LEASt more than -4 opposed to this sign> wich just means greater than

because when you work with variables you usually cannot find the exact amount especially when you are rounding so you know that its biigger than no less than or at least -4

the answer is the arrow going to the right because its not a negative number and a closed circle

please please mark as brainliest

Which of the lists of letters all have line symmetry? A, B, C, D W, X, Y, Z L, M, N, O S, T, U, V

Answers

Answer:

A, W, X, Y, M, O, T, U, V, C, D

Step-by-step explanation:

If you put a line through the middle, then the left and the right side will look the same

ASAP! GIVING BRAINLIEST! Please read the question THEN answer CORRECTLY! NO guessing. I say no guessing because people usually guess on my questions.

Answers

Answer:

B. F(x) = 3(x - 2)² - 2

Step-by-step explanation:

→The function F(x) narrowed, meaning the absolute value being multiplied to the function is greater than 1.

→The function F(x) flipped over the x-axis, this means that the number being multiplied has to be a negative.

→The function F(x) shifted to the left 2 units, this means there needs to be a 2 being added.

→The function F(x) shifted downwards 2 units, meaning there needs to be a 2 being subtracted from the whole function.

This gives us the correct answer of "B. F(x) = 3(x - 2)² - 2."

Solve the equation then write how many solutions there is in this problem: 8x-3+14=24x+5

Answers

Answer:

x = 0.375

Step-by-step explanation:

Step 1: Simplify both sides of the equation

8x − 3 + 14 = 24x + 5

(8x) + (−3 + 14) = 24x + 5

8x + 11 = 24x + 5

- 24

-16x + 11 = 5

-11

-16x = -6

-16x/-16 = -6/-16

x = 3/8

x = 0.375

figure ABCD is a parallelogram what is the perimeter of ABCD

Answers

AB + BC + CD + DA for the perimeter

A rectangle has an area of 96cm2 it's length is 4cm longer than it's width. Calculate the length and width.

Answers

Answer:

I think l

Step-by-step explanation:

first add 96 and4 then 2 I think

In the United States, the mean and standard deviation of adult men's heights are 70 inches (5 feet 10 inches) and 4 inches, respectively. Suppose the American adult men's heights have a normal distribution Whe probability that a randomly chosen American man is taller than 6 feet (72 inches) is equal to:___________. (round off to fourth decimal place, use the given table)
a. 0.6853
b. 0.0062
c. 0.3085
d.0.6915
e. None of these

Answers

Answer:

[tex]P(X>72)=P(\frac{X-\mu}{\sigma}>\frac{72-\mu}{\sigma})=P(Z>\frac{72-70}{4})=P(z>0.5)[/tex]

And we can find this probability using the complement rule and the normal standard table:

[tex]P(z>0.5)=1-P(z<0.5)= 1- 0.69146= 0.3085[/tex]

And the best solution would be:

c. 0.3085

Step-by-step explanation:

For this case we can convert all the values to inches in order to standardize the solution:

[tex] 5ft * \frac{12 in}{1ft}= 60 in[/tex]

[tex] 6ft * \frac{12 in}{1ft}= 72 in[/tex]

Let X the random variable that represent the heights of US mens, and for this case we know the distribution for X is given by:

[tex]X \sim N(70,4)[/tex]  

Where [tex]\mu=70[/tex] and [tex]\sigma=4[/tex]

We are interested on this probability

[tex]P(X>72)[/tex]

We can use the z score formula given by:

[tex]z=\frac{x-\mu}{\sigma}[/tex]

Using this formula we got:

[tex]P(X>72)=P(\frac{X-\mu}{\sigma}>\frac{72-\mu}{\sigma})=P(Z>\frac{72-70}{4})=P(z>0.5)[/tex]

And we can find this probability using the complement rule and the normal standard table:

[tex]P(z>0.5)=1-P(z<0.5)= 1- 0.69146= 0.3085[/tex]

And the best solution would be:

c. 0.3085

Using the normal distribution, it is found that the probability that a randomly chosen American man is taller than 6 feet (72 inches) is equal to:

c. 0.3085

In a normal distribution with mean [tex]\mu[/tex] and standard deviation [tex]\sigma[/tex], the z-score of a measure X is given by:

[tex]Z = \frac{X - \mu}{\sigma}[/tex]

It measures how many standard deviations the measure is from the mean.  After finding the z-score, we look at the z-score table and find the p-value associated with this z-score, which is the percentile of X.

In this problem:

Mean of 70 inches, thus [tex]\mu = 70[/tex].Standard deviation of 4 inches, thus [tex]\sigma = 4[/tex].

The probability of being taller than 72 inches is 1 subtracted by the p-value of Z when X = 72, thus:

[tex]Z = \frac{X - \mu}{\sigma}[/tex]

[tex]Z = \frac{72 - 70}{4}[/tex]

[tex]Z = 0.5[/tex]

[tex]Z = 0.5[/tex] has a p-value of 0.6915.

1 - 0.6915 = 0.3085

Thus, option c.

A similar problem is given at https://brainly.com/question/24855678

There are 390 students at Walker Elementary this year. This is a 30% increase from the previous year. How many students were at Walker Elementary last year?

Answers

Answer:

There were 300 students

Step-by-step explanation:

Original * 30 = increase

Add the increase to get the new number

original + increase = 308

original + original*30% = 390

Factor out original number

original ( 1+30%) = 390

Change to decimal form

original ( 1+.30) = 390

original ( 1.30) = 390

Divide by 1.3

original = 390/1.3

             =300

WILL MARK BRAINLIEST PLEASE HELP

Answers

Answer:

  1) h = -1/2t^2 +10t

  2) h = -1/2(t -10)^2 +72

  3) domain: [0, 20]; range: [0, 50]

Step-by-step explanation:

1.) I find it easiest to start with the vertex form when the vertex is given. The equation of the presumed parabolic path for Firework 1 is ...

  h = a(t -10)^2 +50

To find the value of "a", we must use another point on the graph. (0, 0) works nicely:

  0 = a(0 -10)^2 +50

  -100a = 50 . . . . . . subtract 100a

  a = -1/2 . . . . . . . . . divide by -100

Then the standard-form equation is ...

  h = (-1/2)(t^2 -20t +100) +50

  h = -1/2t^2 +10t

__

2.) The path of Firework 2 is translated upward by 22 units from that of Firework 1.

  h = -1/2(t -10)^2 +72

__

3.) The horizontal extent of the graph for Firework 1 is ...

  domain: 0 ≤ t ≤ 20

The vertical extent of the graph for Firework 1 is ...

  range: 0 ≤ h ≤ 50

If the probability of a machine producing a defective part is 0.05, what is the probability of

finding exactly 5 defective parts from a sample of 100? (Assume that the process follows a

binomial distribution and round answer to four places)

Answers

Answer:

0.1800 to 4 places of decimals.

Step-by-step explanation:

Using the Binomial formula

Probability = 10C5* (0.95)^95 * (0.05)^5

= 100! / 95!*5! *  (0.95)^95 * (0.05)^5

= 0.1800178.

In the rectangular prism, express each of the following in terms of s, t, and u. Give an explanation for each of your answers.
(a) HK
(b) GL
(c)JH

Answers

Complete Question

The complete question is shown on the first  and second uploaded image

Answer:

a

  [tex]\= HK = \= t + \= u[/tex]

b

  [tex]\= GL = \= s - \= t[/tex]

c

[tex]\= JH = \= u + \= s[/tex]

Step-by-step explanation:

Now looking at the diagram

Following the direction of the unit vectors [tex]\= u , \= s, \= t[/tex]

     [tex]\= {HK} = \= {KI} + \= KL[/tex]

=>   [tex]\= HK = \= t + \= u[/tex]

And  

    [tex]\= GL = \= GH + \= GF[/tex]  jjj

=>   [tex]\= GL = \= s - \= t[/tex]

Also

    [tex]\= JH = \= JG + \= JI[/tex]

=>  [tex]\= JH = \= u + \= s[/tex]

Please help! Correct answer only, please! Jason has the following averages in his math class: homework avg: 80 quiz avg: 84 test avg: 74 final exam: 60 if the teacher weights homework at 20%, quizzes at 30%, tests at 40%, and the final exam at 10%, what is jason's class average? A. 74 B. 77 C. 79 D. 82

Answers

Answer:

77

Step-by-step explanation:

80*0.2 + 84*0.3 + 74*0.4 + 60*0.1 = 76.8 = 77

Please help me :( with this

Answers

Answer:

21

Step-by-step explanation:

Similar triangles. MNL is just ABC but 3/4 the size.

x = 8*3/4 = 6

perimeter woudl be 6+6+9 = 21

on solving x/2 +5/3=_1/2 we get x=​

Answers

Step-by-step explanation:

I hope it's correct... Hope this is what you want

Eliminate the variable t from the set of parametric equations. Graph the equation X=5cost Y=5sint Please explain this, I need to know how to do these kinds of equations for my trig final

Answers

Answer:

x^2 + y^2 = 25

Step-by-step explanation:

x = 5 cos t

            cos t = x/5

y = 5 sin t

           sin t = y/5

cos^2 t + sin^2 t = 1

(x/5)^2 + (y/5)^2  = 1

x^2/25 + y^2/25 = 1

(x^2 + y^2)/25 =1

x^2 + y^2 = 25

In a study investigating the effect of car speed on accident severity, 5000 reports of fatal automobile accidents were examined, and the vehicle speed at impact was recorded for each one. For these 5000 accidents, the average speed was 42 mph and the standard deviation was 15 mph. A histogram revealed that the vehicle speed at impact distribution was approximately normal.

a. Roughly what proportion of vehicle speeds were between 27 and 57 mph?

b. Roughly what proportion of vehicle speeds exceeded 57 mph?

Answers

Answer:

(a) Roughly 68% of vehicle speeds were between 27 and 57 mph.

(b) Roughly 16% of vehicle speeds exceeded 57 mph.

Step-by-step explanation:

We are given that in a study investigating the effect of car speed on accident severity, 5000 reports of fatal automobile accidents were examined.

For these 5000 accidents, the average speed was 42 mph and the standard deviation was 15 mph.

Let X = vehicle speed at impact

SO, X ~ Normal([tex](\mu=42,\sigma^{2} = 15^{2}[/tex])

Here, [tex]\mu[/tex] = population average speed = 42 mph

         [tex]\sigma[/tex] = standard deviation = 15 mph

Since, the distribution is approximately normal; so the 68-95-99.7 empirical rule states that;

68% of the data values lies within one standard deviation points.95% of the data values lies within two standard deviation points.99.7% of the data values lies within three standard deviation points.

(a) Since, it is stated above that 68% of the data values lies within one standard deviation points, that means;

68% data values will lie between [ [tex]\mu-\sigma , \mu+\sigma[/tex] ] , i.e;

        [ [tex]\mu-\sigma , \mu+\sigma[/tex] ]  =  [42 + 15 , 42 - 15]

                                 =  [57 , 27]

So, it means that roughly 68% of vehicle speeds were between 27 and 57 mph.

(b) We have observed above that roughly 68% of vehicle speeds were between 27 and 57 mph which leads us to the conclusion that (100% - 68% = 32%) of the data values will be outside this range.

It is stated that of this 32%, half of the data values will be less than 27 mph and half of the data values will be more than 57 mph.

This means that roughly 16% of vehicle speeds exceeded 57 mph.

One day Pat Unger worked for a total of 8
hours. She worked 3 hours more in the after-
noon than she worked in the morning. How
long did she work in the afternoon?

Answers

Answer:

5.5 hours

Step-by-step explanation:

Let the no. of hours worked in morning by Pat = x hours

given that "She worked 3 hours more in the after-

noon than she worked in the morning"

No. of hours worked in Afternoon by Pat = x + 3  hours

Total hours worked in the day = x + x+3 = 2x +3 hours  (1)

It is given that Pat worked for 8 hours that day (2)

thus, using 1 and 2 we have

2x +3  =  8

=>2x = 8 - 3 = 5

=> x = 5/2 = 2.5

no. of hours worked in morning by Pat = x hours = 2.5 hours

No. of hours worked in Afternoon by Pat = x + 3  hours = 2.5 + 3 hours

 No. of hours worked in Afternoon by Pat = 5.5 hours --- Answer.

We are planning on introducing a new internet device that should drastically reduce the amount of viruses on personal computers. We think the price should be $39.99, but are not sure on the percentage of people that would buy it. We do some research and find the following information; Studies from the 1930’s indicate that percentage should be between 30% and 40% Similar products were launched recently at a price of $4,000 and nobody bought it. A nationwide poll on this type of product and price was run earlier this year, with percentages running from 75% to 80%. We are going to conduct an additional focus group before we launch the product. What should the sample size be if we want a 95% CI to be within 5% of the actual value?

Answers

Answer:

The sample size required is 289.

Step-by-step explanation:

Let p be population proportion of people that would buy the product.

It is provided that the nationwide poll on this type of product and price was run earlier this year, with percentages running from 75% to 80%.

Assume that the sample proportion of people that would buy the product is, [tex]\hat p=0.75[/tex].

A 95% Confidence Interval is to be constructed with a margin of error of 5%.

We need to determine the sample size required for the 95% Confidence Interval to be within 5% of the actual value.

The formula to compute the margin of error for a (1 - α)% confidence interval of population proportion is:

[tex]MOE=z_{\alpha/2}\times\sqrt{\frac{\hat p(1-\hat p)}{n}}[/tex]

The critical value of z for 95% confidence interval is,

z = 1.96.

Compute the sample size required as follows:

[tex]MOE=z_{\alpha/2}\times\sqrt{\frac{\hat p(1-\hat p)}{n}}[/tex]

      [tex]n=[\frac{z_{\alpha/2}\ \sqrt{\hat p(1-\hat p)} }{MOE}]^{2}[/tex]

         [tex]=[\frac{1.96\cdot \sqrt{0.75(1-0.75)} }{0.05}]^{2}\\\\=(16.9741)^{2}\\\\=288.12007081\\\\\approx 289[/tex]

Thus, the sample size required is 289.

Other Questions
Please help with my Algebra What is the solution to the system of equations? {6x2y=144x3y=31 If $5a+2b=0$ and $a$ is two less than $b$, what is $7b$? Shakespeares word choice in the passage creates a tone that is 10. What is an expression for the perimeter of the triangle?11. What is an expression for the area of the triangle?12. Think about values for x, y, and z that would make the following statements true. If you think these values exist, give anexample of them. If you think they do not exist, explain why.a.The perimeter and area are both rational numbers.b.The perimeter is an irrational number and the area is a rational number.C.The perimeter and area are both irrational numbers. Hardwig Inc. is considering whether to pursue a restricted or relaxed current asset investment policy. The firm's annual sales are expected to total $3,600,000, its fixed assets turnover ratio equals 4.0, and its debt and common equity are each 50% of total assets. EBIT is $150,000, the interest rate on the firm's debt is 10%, and the tax rate is 40%. If the company follows a restricted policy, its total assets turnover will be 2.5. Under a relaxed policy its total assets turnover will be 2.2. Refer to the data for Hardwig, Inc. Assume now that the company believes that if it adopts a restricted policy, its sales will fall by 15% and EBIT will fall by 10%, but its total assets turnover, debt ratio, interest rate, and tax rate will all remain the same. In this situation, what's the difference between the projected ROEs under the restricted and relaxed policies? a. 2.46% b. 2.98% c. 3.27% d. 2.24% e. 2.70% Lance bought a used car for $5500. He sold it two years later for $2475. What is the percent change of the price? which situation cannot be represented by this expression 13+8 A Ben gave 8 of his bagels to friends. Now he has 13 left. How many bagels did he start with? B Jack bought 8 books. He will buy 13 more. How many books will he buy altogether? C Zoe is reading an article with 13 pages. She has 8 pages left. How many pages has she read? D Caleb swam for 13 minutes. Then he swam for 8 more minutes. For how many minutes did he swim? The cost to purchase a movie from Move Time is 3.00 per movie as well as a 10.00 membership fee. To purchase a movie from Movie City, the membership fee is 30.00 and every downloaded movie is 2.00. How many downloaded movies must be purchased for the monthly price to be the same. (Can someone turn this into an equation for me?) If they are traveling at the same speed, which will have the most kinetic energy?A. A 5-kilogram ballB. A 25-kilogram ballC. A 10-kilogram ballD. A 1-kilogram ball Mark the use of enjambment in this excerpt from Retirement by Henry Timrod. Drag a green arrow to the end of each enjambed line and a stop sign to the end of each end-stopped line. Which is the solution to -3k-2k+12 = 2k-9? Which graph has a slope of ? A coordinate plane with a straight line. The line starts at (negative 5, negative 4) and passes through points at (0, 1) and (4, 5). A coordinate plane with straight line. The line starts at (negative 5, negative 2) and passes through points at (negative 1, 0) and (4, 5). A coordinate plane with straight line. The line starts at (negative 5, negative 1) and passes through points at (negative 2, 0) and (3, 2). A coordinate plane with straight line. The line starts at (negative 4, negative 5) and passes through points at (0, 0) and (4, 5). Identify the diameter if given the radius and the radius if given the diameter of given circles: 7 cm 25 cm 28 cm 35 cm 100 cm 140 cm Radius = 70 cm Radius = 14 cm Diameter = 14 cm Diameter = 50 cm Radius = 70 cm Radius = 14 cm Diameter = 14 cm Diameter = 50 cm What is the average velocity of atoms in 1.00 mol of neon (a monatomic gas) at 465 K? For m, use 0.0202 kg. if 4a + 6 = 12 - 2a what is 6a Based on what you know about Helen and Annie,predict what will happen in upcoming scenes from TheMiracle Worker.Helen's family will have strong doubts aboutAnnie's abilities to teach Helen.Helen will test Annie to see how far she can pushher.0 Annie will lose confidence in her teaching ability.Annie will persevere in teaching Helen. Darnell spends $100 dollars a weekeating out. His sister told him that if hewould reduce this spending by $50 aweek, he could increase his credit cardpayment to $300 per month. How muchwill he save if he takes his sister's advice?ent?How much will Darnell save by increasinghis monthly payment by $200? When did the following countries become independent states?IndiaPakistan BangladeshSri Lanka. Express 84 as a product of prime factors 59:52Which undefined geometric term is described as a two-dimensional set of points that has no beginning or end?